回折格子に関する質問

このQ&Aのポイント
  • 回折格子に関する問題の解説の一部がわからないので質問します。
  • 問題では、回折格子に垂直に白色光をあてた場合の赤色と紫色の光の2次の明線を生じる角度を求めることが求められています。
  • 具体的な計算方法や結果までは示されているが、なぜm=2の場合に相当するのかがわからないとのことです。
回答を見る
  • ベストアンサー

回折格子

こんにちは。高校物理1分野の回折格子に関する問題の解説の一部がわからないので質問します。 問題: 図のように、1cmあたり5000本の割合で線を引いた回折格子に垂直に白色光をあてた。回折光が入射光となす角をΘとするとき、赤色の光と紫色の光の2次の明線を生じる角Θの値はそれぞれいくらか。但し、赤色の光と紫色の光の波長をそれぞれ8×10^3Å、4×10^3Åとする。(1Å=10^-10)      |      |←入射光      |      |      | ____________ |ーーーーーーーーーーー|←回折格子      ・\      ・ \      ・_/\      ・Θ   \←回折光      ・    \      ・     \      ・      \ 解答:dsinΘ=mλの式において、  d=1/5000(cm)=2×10^-4(cm)   =2×10^-4×10^-2=2×10^-6(m)  m=2の場合に相当する。 ( ★←←←←←←なぜですか★) 以下、解答として  赤色の光については、 sin(Θ1)=m(λ1)/d      =2×8×10^3×10^-10/2×10^-6      =0.8 三角比の表から、Θ1=53.1°  紫色の光については、 sin(Θ2)=m(λ2)/d      =2×4×10^3×10^-10/2×10^-6      =0.4 三角比の表から、Θ1=23.6° 質問は <m=2の場合に相当する。( ★←←←←←←なぜですか★)> です。 どうしてもわからないので質問します。よろしくお願いします。

質問者が選んだベストアンサー

  • ベストアンサー
回答No.2

隣り合う格子を通過した光の光路長が、1/2波長ずれると弱めあい、 1波長ずれると強めあい、3/2波長ずれると弱めあい、2波長ず れると強めあい・・・なので、 2次の明線は、2波長ずれる位置にできます。 問題は、2次の明線の位置をθで求めなさいということなので、 m=2を用いて、θを求めればよいわけです。

kyoto1867
質問者

お礼

回答ありがとうございます。 >2次の明線は、2波長ずれる位置にできます。 2次の明線とは、2つめの明線ということなのですね。やっとわかりました。 ありがとうございました。

その他の回答 (1)

回答No.1

単に問題に >2次の明線 とあるからでしょう。

kyoto1867
質問者

お礼

回答ありがとうございます。 上記「この回答への補足」の補足をさせていただきます。 光の回折の回折格子における”明”条件はdsinΘ=mλにおいて、m=0,1,2,3…とありますが、なぜm=2に特定されるかということです。 問題文中の赤または紫の光の波長が 8×10^3×10^-10=8×10-7 4×10^3×10^-10=4×10-7 なので、20×10^-7×sinΘ=m×4×10^-7(mは0,1,2…)と当てはめると 20×10^-7/4×10^-7×sinΘ=m     5×sinΘ=m     0≦sinΘ≦1より、mは5以下。 までは考えてみました。 なぜ”m=2”に特定できるのでしょうか? もう少し解説していただけるとありがたいです。よろしくお願いします。

kyoto1867
質問者

補足

回答ありがとうございます。 >単に問題に >>2次の明線 とあるからでしょう たぶんヒントをいただけたのだと思います。 質問文中 解答:dsinΘ=mλの式において、  d=1/5000(cm)=2×10^-4(cm)   =2×10^-4×10^-2=2×10^-6(m)  m=2の場合に相当する。 ( ★←←←←←←なぜですか★) ということですが、dsinΘ=mλにおいて、dのみが判明している状態(sinΘ、m、λは規定されていない)で、どうしてm=2が特定できるのでしょうか? 基本的なことかもしれませんが、参考書等を読んでも理解できないので質問します。 よろしくお願いします。

関連するQ&A

  • 回折格子

    図は反射型の回折格子である 角θへの回折光が満たす条件式を波長λ、格子定数d、整数mを用いて記せ 図は角αをなして入射した場合である これはどうやって解くんですか?教えてください!

  • 回折格子

    d(sinα+sinβ)=mλ 上の式より、入射角αの値が大きいほど回折角βの値が小さくなりますよね? しかし、実際に実験を行ったところ、αの値を大きくするほど回折光が広がっていきました。つまり、回折角が大きくなっていきました。どういうことなのでしょうか? 純粋に測定ミスなのでしょうか?ご回答お待ちしています。

  • 回折格子、垂直でない入射

    大学1年です。 回折格子に関する実験を行ったのですが、質問があります。 一般に、 「光が格子面に垂直に入射した場合、波長λの成分が進む回折角は、 d sinθ=mλ で与えられる」 かと思います。 これが、垂直でない場合で入射した場合はどうなるのでしょうか? 式や、言葉などで説明していただけたらと思います。 …というのも、スペクトルの分光の実験をしたところ左右の同値になるはずの回折角の差が角度が大きくなるにつれ大きくなっていたのでそこに関して考察できるのではないかと思い、質問させていただきました。 (左右の平均で回折角を求めるように指示はされていますが…) 不備あれば補足します。 よろしくお願いします。

  • 回折格子における次数と波長の関係について

    回折格子における次数と波長の関係について 物理の回折格子に関する問題です。 3.0[cm]あたり4325[本]の溝をつけた回折格子がある。レーザー光源から波長がλ[m]であるレーザー光を、この回折格子に垂直に当てたとき、スクリーンに明線ができた。この明線のうち、中心から数えて3番目の明線では、回折光と入射光のなす角度が60[°]であった。波長λはいくらか。 という問題なのですが、明線の条件式 d*sinθ=mλ d:格子定数、m:次数、λ:波長 から λ=d*sinθ/m とし、次数m=3のとき60[°]という条件を代入して λ=(0.03/4325)*sin60°/3≒2.0*10^(-6)[m] という波長を算出しました。 しかし、解説を見たところ 光路差Δが波長λと等しい回折格子の間隔をdとすると Δ=d*sin60° Δ=λより λ=(0.03/4325)*sin60°≒6.0*10^(-6)[m] となっていました。 なぜこのような結果になるのでしょうか。 明線ができる条件が波長の整数倍ということなら、中心から3番目の明線の次数はm=3となるのではないでしょうか。光路差には3波長分が入っており、それが角度にして60[°]分、光路差ではd*sin60°に相当するのではないでしょうか。もし 3Δ=3λ ∴Δ=λ ということで、角度だけを考えて波長が算出でき、次数は考慮しないのであれば、中心から数えて1番目や2番目における明線の波長は、回折光と入射光のなす角度が3番目とは違うため、同じ波長にならないように思います。どのように考えればよいのでしょうか。ご教授宜しくお願いします。

  • 回折格子

    1cmあたり500本のすじをひいた回折格子にレーザー光を入射させたところ、9.5mはなれたスクリーン上に30cm間隔の明るい点が得られた。 このレーザー光の波長はいくらか。 A…6.3*10^-17m やり方教えてください。

  • 反射型の回折格子について

    溝の断面形状がのこぎり状の反射型の回折格子で、光同士が強めあう条件「d(sinα+sinβ)=mλ」の導出過程がわかりません。 (dは格子定数、αは入射角度、βは反射角度です) d(sinα+sinβ)は光路差だと思うのですが、なぜ光路差がd(sinα+sinβ)になるのかわからないのです。

  • 回折格子のブレーズ角

    いつもお世話になります。 回折格子のブレーズド ホログラフィック グレーティングですとブレーズ角があると思います。 このブレース角でブレーズ波長(回折効率のピーク波長)が決まるとカタログに書いてありました。 ここで質問なのですが、そうますとブレーズ角は物理的に配置する時に意識する角度でなく (ブレーズ角が何度だからこの角度は+ブレーズ角度分増すとか・・・・) あくまでも結果的に選んだ回折格子はこんな角度でしたと把握する程度なのでしょうか? 私は1次光の出射角を知りたい時は、sinα+sinβ=Mmλに代入して求め、ブレーズ角を意識していません。 α:入射角 β:出射角 M:格子周波数[本/mm] m:何次の回折光 λ:入射波長 ブレーズ角がカタログに記載されている意味を教えて下さい。 よろしくお願いします。 以上

  • 反射型回折格子の光路差について

    反射型回折格子の光路差    d(sinβ-sinα)=mλ-(1)    入射角:α    反射角:β    半導体レーザー:λ この(1)の式を使ってCDの格子定数をもとめる実験をしたのですが、私がした実験の条件として垂直入射だから sinα=0になり    Δx=Lλ/d-(2)    明線の間隔:Δx    スクリーンからCDまでの距離:L この(2)の式を使って、CDの格子定数をもとめていきました。しかし、誤差がすごかったので sinα=0 ではおかしいと思いました。 (2)の式から逆算してしていくと入射角αがマイナスの値になるのです。 入射角がマイナスになることはあるのでしょうか?

  • 回折格子 |Δ(d)|、|Δ(θ)|

    回折格子とスクリーンを平行におき、これらに垂直にレーザー光を当てるとスクリーン上に明暗の縞があらわれました。d:格子定数、θ:回折角とすると、 dsinθ=mλによりθからdを決定しました。次数の高い回折像から求めたdのほうが正しい値に近づく。それはd=m・λ/sin(θ)より Δ(d)=-m・λ・cos(θ)・Δ(θ)/(sin(θ))^2=-d^2・Δ(θ)/λ/m すなわち |Δ(d)|=(d^2/λ/m)・|Δ(θ)| mが大きいほうが誤差が小さい。と以前答えをいただいたのですが、何故微分するのか、|Δ(d)|・|Δ(θ)|またこの2つが何を示してるのかわからないので、改めて質問します。教えてください。何を示してるのかを

  • 回折格子げ-22.5°で色分けされないのは?

    分光器を使用したのですが、回折格子が-22.5のときに 色分けされないのはなぜでしょうか? 22.5度の時には入射角と回折角がおなじになり、0次スペクトル になるということはわかるのですが、なぜ22.5°の時は入射角 と回折角が同じになるのかがわかりません。 お願いします。